Une jolie intégrale

Bonjour à tous. L'égalité $\displaystyle\int_{-\infty}^{+\infty}\frac{\mathrm{d}x}{(\mathrm{e}^x-x)^2+\pi^2}=\frac 1{1+\Omega},\ $ avec $\Omega\mathrm{e}^\Omega=1,\ $ semble être connue sous le nom de formule de Victor Adamchik. Il en existe une démonstration par intégration complexe sur MSE. En connaissez-vous une qui fasse appel à des ingrédients plus simples ? Merci !

Réponses

  • Dedekind93: Après avoir cherché un peu sur M.E il ne semble pas connu de méthode réelle pour calculer cette intégrale.

    Une autre intégrale du même type dont il ne semble pas, à ce jour, exister de calcul par des méthodes réelles.
  • Merci FdP pour cette recherche. Bon, je vais essayer de reprendre mes calculs...avec peu d'espoirs du coup !
  • Une idée la fonction a intégré est de la forme $\frac{1}{a^2+b^2}$ peut-être qu’on peut écrire l’intégrale sous la forme d’une intégrale double
    et Fubini
  • Peut-on avoir la référence exacte de l'intégrale initiale dans MSE ? Merci.
  • La page MSE1 renvoie à la page MSE2
  • Mille excuses Chaurien, j'ai eu peur d'utiliser les liens. J'y parviens : mse
  • @gilles benson super ce wikibooks, merci
  • Ce Wiki est bien mais le calcul fait de cette intégrale utilise le théorème des résidus.
  • La fonction $\varphi(x)=\text{e}^x-x$ est croissante sur $[0;+\infty[$ et décroissante sur $]-\infty,0]$.
    On aurait envie de séparer l'intégrale en une somme de deux intégrales et d'appliquer le changement de variable $y=\text{e}^x-x$. (il faudrait récupérer la fonction de Lambert au passage)
  • FDP, si tu réussi avec l'analyse réel alors chapeau. Il y a quand même un espoir avec P, une vue probabiliste ?
    Le 😄 Farceur


  • Si une solution simple "piétonnière" existait elle serait déjà sur MathExchange. Si une solution élémentaire (i.e: sans analyse complexe) existe elle doit être assez sophistiquée.
    Il y a peut-être quelqu'un qui peut nous renseigner sur la difficulté de la question: Olivier Oloa. Il a déjà travaillé sur des intégrales avec des $\pi^2$ au dénominateur si je me souviens bien. B-)-
  • Tu as raison FDP, il s’intéresse à ce genre de trucs https://www.researchgate.net/profile/Olivier-Oloa
    Le 😄 Farceur


  • Déjà j'ai eu du mal à déterminer, ou plutôt localiser, les pôles. Je vois dans la solution de MSE que c'est bien la difficulté. Il y a peu de chances qu'on trouve une solution sans analyse complexe. Mais ce n'est pas grave : il est bon de voir que chaque nouvel outil mathématique permet de résoudre une nouvelle classe de problèmes.

    Ça me rappelle un problème que j'avais posé aux étudiants lorsque j'assurais une préparation d'été au concours HEC d'admission directe pour diplômés. Ce n'était pas une intégrale mais une série.

    Pour $n\in \mathbb{N}^{\ast }$, on définit $x_n$ réel par : $\tan x_{n}=x_{n}$, $n\pi -\frac{\pi }{2}<x_{n}<n\pi +\frac{\pi }{2}$. Démontrer : $\displaystyle \overset{+\infty }{\underset{n=1}{\sum }}\frac{1}{x_{n}^{2}}=\frac{1}{10}$.

    C'est bien plus simple que le problème posé dans ce fil, et il est possible qu'il y ait une solution avec les moyens élémentaires (je l'ignore), mais il se fait très bien par les résidus, et ici les pôles sont évidents.

    Bonne soirée.
    Fr. Ch.
  • Gebrane: Et, je pense qu'il a un compte sur les mathématiques.net. B-)-

    Chaurien: Si une solution "élémentaire" existe elle sera probablement astucieuse. Il faudrait sans doute commencer par trouver une expression de cette intégrale avec des bornes finies. Ce qui n'est déjà pas une mince affaire me semble-t-il.
  • Chaurien : je suis intrigué et intéressé par ton exo... même si on s'éloigne du sujet de ce fil.
  • Merci à tous pour vos réponses. Pour ton problème, Chaurien, il me semble qu'on pourrait peut-être suivre, même si cela promet d'être technique, la méthode exposée dans le magazine Tangente sup numéro 62 pages 14-15-16 sur l'exemple de la somme des inverses des carrés.
  • Se ramener à intégrer sur un intervalle fermé borné n'était pas si difficile:
    $\displaystyle \int_0^1 \bigg(\frac{1}{x\big(\pi^2+(x-\ln x)^2\big)}+\frac{x}{\pi^2x^2+(1+x\ln x)^2}\bigg)dx$.
  • Chaurien pour ton exercice peux-tu nous demontrer que $$\displaystyle \overset{+\infty }{\underset{n=1}{\sum }}\frac{1}{x_{n}^{4}}=\frac{1}{350}$$
    Le 😄 Farceur


  • Pour l'exercice de Chaurien, on trouve une solution ici : https://artofproblemsolving.com/community/c7h182057p3095748
  • @gebrane peux-tu démontrer que $$\sum_{n=1}^{\infty } (-1)^{n-1} \dfrac{\sqrt{1+x_n^2}}{x_n^4}=\dfrac{27}{2800} \quad ?$$
     
  • @bd a condition que tu me démontres d'abord $\displaystyle \overset{+\infty }{\underset{n=1}{\sum }}\frac{1}{x_{n}^{4}}=\frac{1}{350}$
    Les premiers sont les mieux servis
    Le 😄 Farceur


  • gebrane : $\displaystyle \left(\sum_{k=1}^{+\infty} \dfrac{1}{x_k^2}\right)^2 = \left(\sum_{k=1}^{+\infty} \dfrac{1}{x_k^4}\right) + 2\sum_{1\leqslant k_1<k_2} \dfrac{1}{x_{k_1}x_{k_2}}$.
    Or, d'après le lien de AoPS que j'ai posté plus haut : $\displaystyle \sum_{k=1}^{+\infty} \dfrac{1}{x_k^2} = \dfrac{1}{10}$ et $\displaystyle \sum_{1\leqslant k_1<k_2} \dfrac{1}{x_{k_1}x_{k_2}} = \dfrac{1}{280}$. D'où $\displaystyle\sum_{k=1}^{+\infty} \dfrac{1}{x_k^4} = \dfrac{1}{10^2} - \dfrac{2}{280} = \dfrac{1}{350}$.

    De même, en élevant au cube, à la puissance 4, etc. on doit pouvoir calculer $\displaystyle\sum_{k=1}^{+\infty} \dfrac{1}{x_k^{2j}}$ pour tout $j\geqslant 1$.
  • J'ai rappelé ce problème parce que c'est pour moi un exemple d'utilisation du théorème des résidus.
    J'avais fabriqué un énoncé qui définit l'intégrale $J_n,\ n \in \mathbb N^* $, de $f(z)=\dfrac{\cos z}{\sin z - z \cos z}$ sur le cercle $\Gamma_n$ de centre $0$ et de rayon $n \pi$. Il faut prouver que l'équation $\tan z=z$ dans $\mathbb C$ n'a pas de solutions non réelles, ce qui implique que les pôles de la fonction $f$ sont $0$ et les $\pm x_n$. La limite de $J_n$ quand $n \rightarrow + \infty$ donne la somme de la série de terme général $\dfrac 1{x_n^2}$.
    Cette méthode s'adapte pour répondre à la question de Gebrane, et même avec des sommes de puissances paires plus grandes.
    Mais peut-être y a-t-il des solutions qui utilisent d'autres méthodes.
    Bonne journée.
    Fr. Ch.
  • @Gebrane
    Désolé j'ai été grillé. Alors je te laisse prouver mon égalité.
     
  • Bonjour
    @bd qu'on me grille aussi
    @Chaurien, si tu veux t'investir en analyse fonctionnelle, voici une autre méthode qui fait le lien entre tes $x_n$ et les valeurs propres d'un certain opérateur https://www.math.ucdavis.edu/~saito/publications/saito_rayleighfunc.pdf ou bien voir la reponse de Jean Marie dans https://math.stackexchange.com/questions/75206/sum-of-the-squares-of-the-reciprocals-of-the-fixed-points-of-the-tangent-functio
    Le 😄 Farceur


  • Chaurien
    Modifié (12 Feb)
    Ces temps-ci je m'amuse avec les résidus, alors j'ai retrouvé la question posée par bd2017 : $\displaystyle \sum_{n=1}^{+ \infty } (-1)^{n-1} \dfrac{\sqrt{1+x_n^2}}{x_n^4}=\dfrac{27}{2800} $.
    Rappelons que les $x_n$ sont définis, pour $n \in \mathbb N^*$, par : $\tan x_n=x_n$, $n \pi <x_n < n \pi + \frac {\pi}2$.
    Cette question était adressée à Gebrane, mais celui-ci semble avoir pris des vacances, alors je m'y colle.
    En fait, on a : $(-1)^{n} \sqrt{1+x_n^2}=\dfrac 1{\cos x_n}$ (khôlossale finesse !), ce qui nous conduit à considérer la fonction $f(z)=\frac 1{z^2 \cos z (\tan z - z)}=\frac 1{z^2 ( \sin z -z \cos z)}$, dont les pôles sont les $\pm x_n$, simples, et $0$.
    La partie embêtante c'est le calcul du résidu en $0$, qui demande un développement limité de $\sin z - z\cos z$ à la précision $o(z^7)$, et l'on trouve $\textrm{Res}(f,0)= \frac {27}{1400}$. C'est bon signe.
    Le résidu en un pôle simple $\alpha \neq 0$ est, sans surprise : $\textrm{Res}(f,\alpha)= \frac 1{\alpha^4 \cos \alpha}$.
    Si l'on définit toujours le cercle $ \Gamma _{n}=\{z~|~\left\vert z\right\vert =n\pi \}$, pour $n \in \mathbb N^*$, alors : $\displaystyle \frac{1}{2\pi i}\int_{\Gamma _{n}}f(z)dz=\textrm{Res}(f,0)+2 \underset{k=1}{\overset{n-1}{\sum }} \textrm{Res}(f,x_k )$.
    J'ai fait observer par ailleurs que pour $|z| = n \pi$ on a : $|\cos z| \ge 1$ et $|\tan z| < 2$.
    On en déduit : $\displaystyle \lim_{n \rightarrow + \infty} \int_{\Gamma _{n}}f(z)dz=0$. La formule proposée en découle.
    Bonne journée.
    Fr. Ch.
    16/07/2021
  • Bonjour
    @Chaurien J'ai un peu oublié mais il me semble que j'avais trouvé la formule 2 façons, une avec les résidus et une avec les valeurs propres de l'opérateur laplacien.
    Une question que je me pose : la somme de la série $\sum \dfrac{(-1)^n}{x_n}$ est-elle calculable avec une de ces 2 méthodes. A priori cela me semble difficile...
     
Connectez-vous ou Inscrivez-vous pour répondre.